Miguel did the following math problem: 6^5 ∙ 6^4 = 36^9.

Which of the following is a true statement?

He is correct.
He is incorrect because he should have subtracted the exponents.
He is incorrect because he should have multiplied the exponents.
He is incorrect because he multiplied the bases.

Answers

Answer 1

Answer:

He is incorrect because he multiplied the bases.

Step-by-step explanation:

[tex] \huge 6^5 ∙ 6^4 = {6}^{5 + 4} = {6}^{9} [/tex]

He is incorrect because he multiplied the bases.


Related Questions

I’ll mark you brainlist I’ll mark you brainlist write in y=mx+b form

Answers

y=-2/1x ivigvyvitvtuyvsybsubeeuwub

Write the equation in the graph below.

Answers

Answer:

Y=1/2x+2

Step-by-step explanation:

The slope is always rise/run and the Y intercept is 2.

Write a conjecture that describes the pattern below.

3, 9, 27, 81...

Answers

Answer:

the pattern is going by 3s and u would multiply 3 by each number

A teacher has 600 counters. He keeps 96 counters at his desk and gives the rest to 3 groups of students. The first group of students gets twice as many counters as the second group of students. The third group of students gets four times as many counters as the second group. How many counters did the second group get?

Answers

Answer:

o

Step-by-step explanation:

o

if pml is congruent to trq find qr?

Answers

Answer:

16

Step-by-step explanation:

what is the product of (3x+5) and (x+4)?

Answers

Answer:

3x^2+17x+20

Step-by-step explanation:

Answer: x = y

y = -4

y = 3x + 2

x = 8

Simplify: (-3a2b5)2

Find (x - 4)2`Find the product: (3x + 5)(4x - 1)

Use FOIL:

F: 3x*4x=12x^2

O: 3x*-1 = -3x

I: 5*4x = 20x

L: 5*-1 = -5

Combine these products to get:

12x^2+17x-5

------------------------

Which of the following is the equation of a vertical line?

x = y

y = -4

y = 3x + 2

x = 8

Answer x=8 ; The graph is the set of points where x=8.

-----------------------

Simplify: (-3a2b5)2

=(-3a^2b^5)(-3a2b^5)

=9a^4b^10

------------

Find (x - 4)^2

=(x-4)(x-4)

Use FOIL to get:

x^2-4x-4x+16

=x^2-8x+16

Step-by-step explanation:All work needed to arrive at a correct answer. If it is not a question that requires work, you will need to provide a complete explanation of how you determined the correct answer. (Explanations for those problems not requiring work must demonstrate your understanding of the concept.)

Which statement is true about this quadratic equation? y=-2x^2+9x-12 A. There are two real solutions. B. There is one complex solution. C. There are two complex solutions. D. There is one real solution.

Answers

A quadratic equation is an equation that can be placed in the form a·x² + b·x + c = 0

The option that gives the true statement about the quadratic equation is option C.

C. There are two complex solutions

Reason:

The given quadratic equation can be presented as follows;

y = -2·x² + 9·x - 12

The factored form of the quadratic equation is presented as follows;

The radical of the given quadratic equation is therefore;

[tex]\sqrt{9^2 - 4 \times (-2) \times (-12)} = \sqrt{81 - 96} = \sqrt{-15}[/tex], which gives the following solutions;

[tex]x = \dfrac{(-9\pm\sqrt{-15} )}{2 \times (-2)} = \dfrac{-9 \pm \sqrt{-15} }{-4}[/tex]

[tex]x = \dfrac{-9 +\sqrt{-15} }{-4} \ or \ x = \dfrac{-9 -\sqrt{-15} }{-4}[/tex]

Therefore there are two complex solutions;

Learn more about quadratic equations here:

https://brainly.com/question/12402430

Answer:

the answer is c

Step-by-step explanation:

There are two complex solutions.

At the gas station, each liter of gas costs \$3$3dollar sign, 3, but there's a promotion that for every beverage you purchase you save \$0.20$0.20dollar sign, 0, point, 20 on gas.
Is your total savings on gas proportional to the number of beverages you purchase?

Yes Or No

Answers

Answer:

Yes, your total savings on gas is proportional to the number of beverages you purchase.

Step-by-step explanation:

What is a proportional relationship?

A proportional relationship, also known as direct relation, between two variables exists when the ratio between every pair of data is constant.

In this case the variables are:

number of beverage purchased

savings

Thus, as per every beverage purchase you save $ 0.20 the ratio of total savings to the number of beverage purchases is:

savings / beverages purchase = $ 0.20 / beverage

That means that you can determine the savings by multipliying the number of beverages times $ 0.20.

For proportional relationships it is easy to build a table that shows the behavior of the variables. This shows you that:

Number of beverages   savings    ratio

        1                              $ 0.20      $0.20 / 1 beverage = 0.20

        2                             $ 0.40      $0.40 / 2 beverages = 0.20

        3                             $ 0.60       $0.60 / 3 beverages = 0.20

        4                             $ 0.80       $0.80 / 4 beverages = 0.20

What is Зх — 4y + 3x + 2y

Answers

Answer:

6x - 2y

Step-by-step explanation:

3x - 4y + 3x + 2y

combine like terms

6x -2y

hope this helps!!!

Simply: 6x-2y

3x-4y+3x+2y

On a sunny day, if a 36-inch yardstick casts a 21-inch shadow,
how tall is a building whose shadow is 168 ft?

Answers

It would be 288 ft.

what 30+30+30+4+60+9+80

Answers

Answer:

243

Step-by-step explanation:

please please help i’ll give brainliest tysm:))

Answers

Answer:

The answer is 66 feet.

Hope it helps!!

HURRY HELP WILL GIVE MORE POINTS after answered

Answers

HL
the hypotenuses are congruent and one side of the triangle is congruent

Kaylee earned $446.40 at her job when she worked for 18 hours. What was her hourly wage, in dollars per hour?

On the double number line below, fill in the given values, then use multiplication or division to find the missing value.

Answers

Answer:

Kaylee earned $24.8 per minute.

Step-by-step explanation:

Basically, just divide the total wage, by hours earned.

in the blanks I assume you just put 24.8 dolars per 1 hour and then the final 446.40 for 18 hours at the end of the number line :)

If possible may I have brainlist? Thanks g

marrisa dance studio charge a monthly fee

Answers

Answer:

lool what's the question

Step-by-step explanation:

how is life treating you?

What are the zeroes of the function?

y = (x – 3)(x + 2)(x – 2)
A. –3, 2, –2
B. –3, 2, 2
C. 3, –2, 2
D. 3, 2, 2

Answers

Answer:

Option C

Step-by-step explanation:

x - 3 = 0

Add 3 to both sides;

x = 3

x + 2 = 0

Subtract 2 from both sides;

x = -2

x - 2 = 0

Add 2 to both sides;

x = 2

Which inequality correctly compares Three-fourths, One-seventh, and Five-sixths? One-seventh < Three-fourths < Five-sixths One-seventh < Five-sixths < Three-fourths Three-fourths < Five-sixths < One-seventh Three-fourths < One-seventh < Five-sixths

Answers

Answer:

The inequality which correctly compares Three-fourths, One-seventh, and Five-sixths is : One-seventh < Three-fourths < Five-sixths.

The first option is correct.

Step-by-step explanation:

To determine which inequality correctly compares Three-fourths, One-seventh, and Five-sixths, we will arrange them in ascending order.

To do this, first, we will determine the LCM of the fractions denominators..

Three-fourths, One-seventh, and Five-sixths

[tex]\frac{3}{4}, \frac{1}{7}, and \frac{5}{6}[/tex]

The denominators are 4, 7 and 6.

LCM of 4,7 and 6 is 84

Then, we will write the fractions in a form whereby they all have the same denominators.

[tex]\frac{3}{4}, \frac{1}{7}, and \frac{5}{6}[/tex]  can be written as

[tex]\frac{63}{84}, \frac{12}{84}, and \frac{70}{84}[/tex]

The resulting order of the numerators gives the order of the fractions.

Now, we will arrange [tex]\frac{63}{84}, \frac{12}{84}, and \frac{70}{84}[/tex] in ascending order. This becomes

[tex]\frac{12}{84}, \frac{63}{84},and \frac{70}{84}[/tex].

∴ The ascending order of the fractions is [tex]\frac{1}{7}, \frac{3}{4},and \frac{5}{6}[/tex].

That is, One-seventh, Three-fourths, and Five-sixths.

Hence, we can write that

One-seventh < Three-fourths < Five-sixths.

This is the inequality which correctly compares Three-fourths, One-seventh, and Five-sixths

Answer:

d

Step-by-step explanation:

Bill spent less than $36 on Kitten and puppy magazines kitten magazines cost $4 each and puppy magazines cost $3 each A) write an inequality that describes the situation B) list two possible combinations of puppy and kitten magazines C) if bill bout 3 kitten magazines, how many puppy magazines can he buy?

Answers

Answer:

A) 4k + 3p < 36

B) 3 kitten magazines and 4 puppy magazines

4 kitten magazines and 6 puppy magazines

C) If bill bought 3 kitten magazines, then he buys less than 8 puppy magazines.

Step-by-step explanation:

Let:

The number of kitten magazines = k

The number of puppy magazines = p

A) Write an inequality that describes the situation

We are told:

Bill spent less than $36 on Kitten and puppy magazines kitten magazines cost $4 each and puppy magazines cost $3 each

Hence:

$4 × k + $3 × p < 36

4k + 3p < 36

B) List two possible combinations of puppy and kitten magazines

4k + 3p < 36

Bill can buy:

3 kitten magazines and 4 puppy magazines

4 kitten magazines and 6 puppy magazines

C) If bill bought 3 kitten magazines, how many puppy magazines can he buy?

Using the inequality above:

4k + 3p < 36

4k + 3p < 36

4 × 3 + 3p < 36

12 + 3p < 36

3p < 36 - 12

3p < 24

p < 24/3

p < 8

If bill bought 3 kitten magazines, then he buys less than 8 puppy magazines.

4cos^2(x)-7cos(x)+3
Please help and explain how you did it :)

Answers

I assume you're asked to solve

4 cos²(x) - 7 cos(x) + 3 = 0

Factor the left side:

(4 cos(x) - 3) (cos(x) - 1) = 0

Then either

4 cos(x) - 3 = 0   or   cos(x) - 1 = 0

cos(x) = 3/4   or   cos(x) = 1

From the first case, we get

x = cos⁻¹(3/4) + 2  or   x = -cos⁻¹(3/4) + 2

and from the second,

x =

where n is any integer.

Which value is the solution to the equation - 8 = 6-7x?
A. -7
B. - 2
c. 2
D. 5

Answers

Answer:

-8=6-7x (subtract 6 from both sides)

-14=-7x (divide each side by -7)

x=2

the answer is C

Answer:

c: 2

Step-by-step explanation:

-8 = 6 -7x, isolate the -7 by moving the 6 to the left side of the equation so it becomes -8-6 = -7x --> -14 = -7x and then divide by both sides by -7 and then you get x =2, you can plug in the number back into the equation to make sure

1/2x-3=9 please bhelp me

Answers

Answer:

x=24

Step-by-step explanation:

Step 1: Add 3 to both sides: 1/2x-3+3=9+3

Simplify that: 1/2x=12

Step 2: Multiply both sides by 2: 1/2x*2=2*12

Simplify: x=24

Hope this helped! :)

The answer is X=24 because you have you would have to simplify

dudee plz help will be given the brainlist if answered correctly

Answers

Answer:

hey im not actually here to help you, i just need points. thanks for the points

Step-by-step explanation:

The speed of a stream is 3 km per hour. A boat travels upstream 12 km and then returns to its original position downstream along the same route. If the speed of the boat in still water is 9 km per hour, what is the average speed of the boat for the entire round trip?

Answers

Answer:

7 km

Step-by-step explanation:

let the speed of boat in still water is x km/hr. 

given speed of stream is 3 km/hr. 

then speed of boat upstream = ( x - 3) km/hr

speed of boat downstream = (x + 3 ) km/hr 

Distance travelled upstream = 4 km

time taken to travel upstream = distance travelled upstream / speed of boat 

upstream

= 4 / (x - 3 ) .............(1) 

Distance travelled downstream = 10 km

time taken to travel downstream = distance travelled downstream / speed of boat 

downstream

= 10 / (x + 3) ............(2)

given that boat takes same time to travel upstream and downstream.i.e; (1) & (2) are equal

i.e. 4/(x-3) = 10/ (x + 3 ) 

4 * (x +3 ) = 10 * (x- 3)

4x + 12 = 10x - 30

12 + 30 = 10x -4x 

42 = 6x 

x = 42 / 6 = 7. 

i.e speed of boat in still water is 7 km/hr.

A rate in which the second quantity in the comparison is one unit is known as a
______________________.

Answers

Answer:

Unit rate

Step-by-step explanation:

Answer:

Unit rate

Step-by-step explanation:

Unit rate -- A rate in which the second quantity in the comparison is one unit.

what is the equation of a line that is parallel to the x-axis and is 3 units below the x-axis

A.x=3
B.x=-3
C.y=3
D.y=-3​

Answers

Iupjbojhijnjjbvuvchjvujvubvvvcuvcuvfubcvcgggggvujb

this is duee in 1 hourrr helllp!!!!

Answers

Answer:

4/5

Step-by-step explanation:

Rise = 4

Run = 5

~theLocoCoco

Answer:

4/5

Step-by-step explanation:


The function y - 3 = 4(x + 2) is in point-slope form. Select the equivalent equation in slope-intercept form

Answers

Answer:

All we need to do is solve for y.

[tex]y - 3 = 4(x + 2)\\\\y - 3 = 4x + 8\\\\y = 4x + 11[/tex]

The answer is A. y = 4x + 11.

What is 1/3 + 4/9 and how to solve

Answers

Make them same denominator
1/3 * 3/3 = 3/9
Now add them
3/9 + 4/9 = 7/9

Answer:

7/9

Step-by-step explanation:

To add fractions, you have to have like denominators. First find the LCM (Least Common Multiple) of both denominators. That is 9. The second fraction already has a denominator of 9, so leave it as is. The first fraction doesn't so what will happen is you will be converting it into a fraction with a denominator of 9. Multiply the denominator by 3 to get 9. Then multiply the top by 3 because whatever you do to the bottom you do the top. You will then get 3/9. 3/9+4/9 is equal to 7/9 because you just add the numerators, but keep the denominators the same.

what is π plus e its so easy

Answers

It should be 5.85828182846 hope this helps

The newspaper in Haventown had a circulation of 80,000 papers in the year 2000. In 2010, the circulation was 50,000. With x=0 representing the year 2000, the graph below models this scenario. What number will complete the point-slope equation that models this scenario?

y−50,000=[_______](x−10)

Answers

Answer:

Step-by-step explanation:

sup

Other Questions
photo attached, what structures are A-F please help watch out for hihisusume that will answer your question and not actually give an answer that are only doing it so they can get the points Which of the following correctly represents the areas controlled by European countries on the map? You have been working hard and saving money to buy your first used car - woo hoo! But you still need more money. (It's an expensive car!) You borrow $10,000 from the bank at a 9% rate for 5 years. How much interest will you pay on this loan? Write your answer as a number with no dollar sign. Why do scientists believe that a human can more quickly crack the code for protein design than a computer? As a conservative, who did Nixon appeal to? *A-Liberals who supported the Great Society programs.B-Civil Rights activists who sought to advance integration demands for equality.C-The New Left and student activists who challenged the government to take moreradical steps to improve race relations and end the war in Vietnam.D- A "Silent Majority" who resented the tension of the 1960s social change andmissed the "simplicity" of the 1950s. write a letter to your friend describing him/her about your country nepalGuys plz help me with this question write a letter about nepal. If u guys help me with this question i will make you brainliest and give 25 points. But its so urgent so plz do it fast. in mrs. smiths class every time you turn in assignments you earn 1 homework pass. write 3 equivalent ratios of turned in assignments to homework passeshelp please I need it Why did the Mongols use a Chinese style government? 2m+11=-73 what number does m=? Andrew has already played 32 minutes on his video game today and wants to play an additional 15 minutes each of x games after school today. What equation can be used to find y, the total minutes Andrew will play video games today? Can somebody that is good at science help me with these two questions? Please and thank you! help please I don't know it Which of the following statements about bonds is TRUE? A. A bond represents a share of ownership in a company, B. Bonds are considered relatively risky compared to stocks. C. Bonds pay out annual dividends. D. When a bond matures you get the full amount you loaned back with interest. Thomas needed a loan to buy a car. He borrowed $24,000 from the bank at a rate of 8.25% simple interest for 36 months. What was the total amount that Thomas had to pay back? I WILL GIVE THE BRAINLYISTanswer quickly please segregate (v.)Definition: to separate and treat unfairlySynonyms: divide, split apart, isolateAntonyms: integrate, bring togetherWhich two words are antonyms of the verb segregate? jointear apartuniteopenchase Thirteen less than four times a number is 17 CAN SOMEONE PLEASE HELP ME?? Which scientist discovered DNA after experimenting with white blood cells? A: Watson B: Franklin C: Miescher D: Wilkins